0

Linear and synthetic method of division - class-IX

Description: linear and synthetic method of division
Number of Questions: 64
Created by:
Tags: remainder and factor theorems polynomials and factorization multiplication and division of algebraic expressions maths polynomials basic algebra
Attempted 0/64 Correct 0 Score 0

If $P(x)$ and $Q(x)$ are two polynomial such that $f(x)=P(x^3)+Q(x^3)$ is divisible by $x^2+x+1$, then?

  1. $P(x)$ is divisible by $(z-1)$ by $Q(x)$ is not divisible by $(x-1)$

  2. $Q(x)$ is divisible by $(x-1)$ but $P(x)$ is not divisible by $(x-1)$

  3. Both $P(x)$ and $Q(x)$ are divisible by $(x-1)$

  4. $f(x)$ is divisible by $(x-1)$


Correct Option: C

What is the degree of the remainder atmost, when a fourth degree polynomial is divided by a quadratic polynomial?

  1. $2$

  2. $0$

  3. $4$

  4. $1$


Correct Option: D
Explanation:
Here $f(x)$ represent dividend and $g(x)$ represent division

$g(x)=$ quadratic polynomial $=ax^2+bx+c$

$\therefore deg(g(x))=2$, $deg(f(x))=4$

quotient $q(x)$ is of degree $=2$ $(=4-2)$

Remainder $R(x)=$ degree $1$ or less than $1$.

Can $(x - 1)$ be the remainder on division of a polynomial $p(x)$ by $2x + 3$?

  1. Yes

  2. No

  3. Cannot be justified

  4. None of above


Correct Option: B
Explanation:

Division algorithm stated that a polynomial $f(x)$ can written as
   $f(x) = g(x)q + r$     where $q$ and $r$ are unique integer and $0 <= r < g(x)$.
Here,
$g(x)=2x+3$ and $r(x)=x-1$
The power of the remainder is always less than the power of the divisor
Here, the degree of remainder is $1$ and the degree of divisor is $1$, which is not possible. Thus, $(x-1)$ cannot be the remainder of $p(x)$ when divided by $(2x+3)$

If quotient = $3x^2\, -\, 2x\, +\, 1$, remainder = $2x - 5$ and divisor  = $x + 2$, then the dividend is:

  1. $3x^3\, -\,4x^2\, +\, x\, -\, 3$

  2. $3x^3\, -\, 4x^2\, -\, x\, +\, 3$

  3. $3x^3\, +\, 4x^2\, -\, x\, +\, 3$

  4. $3x^3\, +\, 4x^2\, -\, x\, -\,3$


Correct Option: D
Explanation:

Dividend = (divisor $\times$ quotient) + remainder
= $(3x^2\, -\, 2x\, +\, 1)\, \times\,  (x\, +\, 2)\,+\, (2x\, -\, 5)$
= $3x^3\, +\, 4x^2\, -\, x\,-\, 3$

The remainder if $a{x}^{3}+b{x}^{2}+cx+d$ is divided by $ax+b$

  1. $ad-bc$

  2. $\cfrac{1}{a}(ad-bc)$

  3. $\cfrac{a-bc}{d}$

  4. $\cfrac{a+b+cd}{2}$


Correct Option: B
Explanation:

Here, $p(x) = a{x}^{3}+b{x}^{2}+cx+d$ and factor of $ax+b$ is
$ax+b = 0$
$x = -\frac ba$
$ p(-\frac ba) = a(-\frac ba)^{3}+b(-\frac ba)^{2}+c(-\frac ba)+d$
$p(-\frac ba) = -\frac {b^3}{a^2}+ \frac {b^3}{a^2} -\frac {bc}{a}+d $
$p(-\frac ba) =  -\frac {bc}{a}+d $
$p(-\frac ba) = \frac 1a (ad - bc) $
when $a{x}^{3}+b{x}^{2}+cx+d$ is divided by $ax+b$ then remainder is $p(-\frac ba) = \frac 1a (ad - bc) $
Option B is correct.

If $ \displaystyle 2x^{3}+4x^{2}+2ax+b $  is exactly divisible by $ \displaystyle x^{2}-1 $  Then the value of $a$ and $b$ respectively will be 

  1. $1,2$

  2. $-1,4$

  3. $1,-2$

  4. $-1,-4$


Correct Option: D
Explanation:

Since $\displaystyle f\left ( x \right )=2x^{3}+4x^{2}+2ax+b$ is exactly divisible
by $x\displaystyle ^{2}-1=\left ( x-1 \right )\left ( x+1 \right )$
$\displaystyle \therefore f\left ( 1 \right )=0$ and $\displaystyle f\left ( -1 \right )=0$
These give
    $2+4+2a+b=0$
or $2a+b+6=0$          .....(i)
and $-2+4-2a+b=0$
or $2a-b-2=0$        ....(ii)
Solving equations (i) and (ii) we get 
$a=-1, b=-4$

The product of $x^2y$ and $\cfrac{x}{y}$ is equal to the quotient obtained when $x^2$ is divided by ____.

  1. $0$

  2. $1$

  3. $x$

  4. $\cfrac{1}{x}$


Correct Option: D
Explanation:

$x^2y\times\dfrac{x}{y}=x^3\Rightarrow $when $x^2$ is divided by$ \dfrac{1}{x}\ $gives $  x^3$

There is a remainder of 3 when a number is divided by 6. What will be the remainder if the square of the same number is divided by 6?

  1. 1

  2. 0

  3. 3

  4. 2


Correct Option: C
Explanation:

Let the no. be $'x'$ $\Rightarrow x=6q+3$

$x^{2}=(6q+3)^{2}=36q^{2}+36q+9$
Divide $x^{2}$ by $6$ 
we get $\boxed{x^{2}=6(6q^2+6q+1)+3}$
Hence the remainder is $\boxed{3}$

If on dividing a non-zero polynomial $p(x)$ by a polynomial $g (x)$, the remainder is zero, what is the relation between the degrees of $p(x)$ and $g (x)$?

  1. degree of $g (x) \ge$ degree of $p(x)$

  2. degree of $g(x) \le$ degree of $p(x)$

  3. degree of $g (x) =$ degree of $p(x)$

  4. Can't say


Correct Option: B
Explanation:

deg $p(x)=$ deg $g(x)+r(c)$

Then, deg $p(x) \ge$ deg $g(x)$

If the polynomial $x^3-x^2+x-1$ is divided by $x-1$, then the quotient is :

  1. $x^2-1$

  2. $x^2+1$

  3. $x^2-x+1$

  4. $x^2+x+1$


Correct Option: B
Explanation:

Divide $x^3-x^2+x-1$ by   $x-1$


         $x-1$ $\overline{)x^3-x^2+x-1(}$  $x^2+1$

                $-(x^3-x^2)$
                   $\overline{\quad\quad\quad\quad+x-1}$
                                   $-(x-1)$
                                   $\overline{\quad\quad\quad0}$

Hence, $B$ is correct.

Find the reminder when ${x^3} + 3{x^2} + 3x + 1$ is divided by $x + \pi $

  1. $\pi$

  2. $-{\pi}^{2}+3{\pi}^{3}-3\pi+1$

  3. $-{\pi}^{3}+3{\pi}^{2}-3\pi+1$

  4. None of these


Correct Option: C
Explanation:
The remainder theorem states that when a polynomial, $f\left(x\right)$, is divided by a linear polynomial , $x - a$, the remainder of that division will be equivalent to $f\left(a\right)$.

Given:$f\left(x\right)={x}^{3}+3{x}^{2}+3x+1$

$f\left(x\right)$ is divided by a linear polynomial , $x+\pi$, the remainder of that division will be equivalent to $f\left(-\pi\right)$.

Remainder$=f\left(-\pi\right)={\left(-\pi\right)}^{3}+3{\left(-\pi\right)}^{2}+3\left(-\pi\right)+1=-{\pi}^{3}+3{\pi}^{2}-3\pi+1$

When the polynomial  ${x^4} + {x^2} + 1$   is divided by $(x + 1)({x^2} - x + 1)$ then the remainder is $ax + b$ , then  $a + b$ is equal to 

  1. $1$

  2. $2$

  3. $3$

  4. $4$


Correct Option: A
Explanation:
$\dfrac{{x}^{4}+{x}^{2}+1}{\left(x+1\right)\left({x}^{2}-x+1\right)}$

$=\dfrac{{x}^{4}+2{x}^{2}+1-{x}^{2}}{\left(x+1\right)\left({x}^{2}-x+1\right)}$

$=\dfrac{{\left({x}^{2}+1\right)}^{2}-{x}^{2}}{\left(x+1\right)\left({x}^{2}-x+1\right)}$

$=\dfrac{\left({x}^{2}-x+1\right)\left({x}^{2}+x+1\right)}{\left(x+1\right)\left({x}^{2}-x+1\right)}$

$=\dfrac{\left({x}^{2}+x+1\right)}{\left(x+1\right)}$

$=\dfrac{x\left(x+1\right)+1}{\left(x+1\right)}$

$=x+\dfrac{1}{x+1}$

Remainder$=1$ is of the form $ax+b$

$\Rightarrow\,a=0,\,b=1$

$\therefore\,a+b=0+1=1$

Find the quotient $q(x)$ and remainder $r(x)$ of the following when $f(x)$ is divided by $g(x)$.
$p(x)=x^3-3x^2-x+3$;
$g(x)=x^2-4x+3$

  1. $q(x)=x-1$ and $r(x)=0$

  2. $q(x)=x+1$ and $r(x)=0$

  3. $q(x)=2x+1$ and $r(x)=0$

  4. $q(x)=2x-1$ and $r(x)=0$


Correct Option: B
Explanation:

Consider the polynomial $f(x)=x^3-3x^2-x+3$ and factorise it as follows:


$f(x)=x^3-3x^2-x+3=x^2(x-3)-1(x-3)=(x^2-1)(x+3)=(x+1)(x-1)(x-3)$

Therefore, $f(x)=(x+1)(x-1)(x-3)$

Now consider the polynomial $g(x)=x^2-4x+3$ and factorise it as follows:

$g(x)=x^2-4x+3=x^2-3x-x+3=x(x-3)-1(x-3)=(x-1)(x-3)$

Therefore, $g(x)=(x-1)(x-3)$


Now divide $f(x)$ by $g(x)$ to get $q(x)$:

$q(x)=\frac { (x-1)(x+1)(x-3) }{ (x-1)(x-3) } =x+1$

Since $f(x)$ is divisible by $g(x)$, therefore, the remainder $r(x)=0$.

Hence, the quotient $q(x)=x+1$ and remainder $r(x)=0$.

Find the quotient $q(x)$ and remainder $r(x)$ of the following when $f(x)$ is divided by $g(x)$.
$p(x)=x^6+x^4-x^2-1$;
$g(x)=x^3-x^2+x-1$

  1. $q(x)=2x^3+x^2+x-1$ and $r(x)=0$

  2. $q(x)=x^3+x^2+x-1$ and $r(x)=0$

  3. $q(x)=2x^3+x^3+x+1$ and $r(x)=0$

  4. $q(x)=x^3+x^2+x+1$ and $r(x)=0$


Correct Option: D
Explanation:

Consider the polynomial $f(x)=x^6+x^4-x^2-1$ and factorise it as follows:


$f(x)=x^6+x^4-x^2-1=x^4(x^2+1)-1(x^2+1)=(x^4-1)(x^2+1)=(x^2-1)(x^2+1)(x^2+1)$
$=(x-1)(x+1)(x^2+1)^2$

Therefore, $f(x)=(x-1)(x+1)(x^2+1)^2$

Now consider the polynomial $g(x)=x^3-x^2+x-1$ and factorise it as follows:

$g(x)=x^3-x^2+x-1=x^2(x-1)+1(x-1)=(x^2+1)(x-1)$

Therefore, $g(x)=(x-1)(x^2+1)$


Now divide $f(x)$ by $g(x)$ to get $q(x)$:

$q(x)=\frac { (x-1)(x+1)(x^{ 2 }+1)^{ 2 } }{ (x-1)(x^{ 2 }+1) } =(x+1)(x^{ 2 }+1)=x^{ 3 }+x^{ 2 }+x+1$

Since $f(x)$ is divisible by $g(x)$, therefore, the remainder $r(x)=0$.

Hence, the quotient $q(x)=x^{ 3 }+x^{ 2 }+x+1$ and remainder $r(x)=0$.


Check whether $g(y)$ is a factor of $f(y)$ by applying the division algorithm.
$f(y)=3y^4+5y^3-7y^2+2y+2$
$ g(y)=y^2+3y+1$

  1. Yes

  2. No

  3. Ambiguous

  4. Data insufficient


Correct Option: A
Explanation:

Given f(y)=$3y^{4}+5y^{3}-7y^{2}+2y+2$ and (g)=$ y^{2}-4y+2$

Then $ y^{2}+3y+1\div 3y^{4}+5y^{3}-7y^{2}+2y+2\setminus 3y^{2}-y+7$
                                   $3y^{4}+9y^{3}+3y^{2}$
                                      -          -           -
                                  ---------------------------------------
                                       $-4y^{3}-10y^{2}+2y+2$
                                        $-4y^{3}-12y^{2}-4y+2$
                                            +          +           +
                                     -----------------------------------------
                                            $2y^{2}+6y+2$
                                             $2y^{2}+6y+2$
                                                -          -       -
                                    --------------------------------------------
                                                            0
So g(Y) is a factor of f(y).                     
                                             

Divide the polynomial $p(x)$ by the polynomial $g(x)$ and find the quotient and remainder. 
$p(x)=x^4-3x^2+4x+5$
$g(x)=x^2+1-x$

  1. $q(x)=x^2+x-3$ and $r(x)=-8$

  2. $q(x)=x^2-x+3$ and $r(x)=8$

  3. $q(x)=x^2+x-3$ and $r(x)=8$

  4. $q(x)=x^2-x-3$ and $r(x)=-8$


Correct Option: C
Explanation:

$x^2-x+1)\overline {x^4-3x^2+4x+5}$ ( $x^2+x-3$
                  $\underline {\underset {-}{}x^4\underset {-}{+}x^2              \underset{+}{-}x^3}$
                  $x^3-4x^2+4x+5$
                  $\underline {\underset {-}x^3\underset {+}{-}x^2\underset {-}{+}x}$
                  $-3x^2+3x+5$
                  $\underline {\underset {+}{-}3x^2\underset {-}{+}3x\underset {+}{-}3}$
                                  $8$
Hence, Quotient=$x^2+x-3$
Remainder=8.

Polynomials $p(x), g(x), q(x)$ and $r(x)$, which satisfy the division algorithm and deg $r(x)=0$, are

  1. $p(x)=x^2+x; g(x)=x+1$;
    $q(x)=5; r(x)=7$.

  2. $p(x)=2x^2+x+1; g(x)=x+3$;
    $q(x)=2x; r(x)=7x$.

  3. $p(x)=x^3+x+5; g(x)=x^2+1$;
    $q(x)=x; r(x)=5$.

  4. None of these


Correct Option: C
Explanation:

option (A) and have deg $r(x)=0$,

in this question $p(x)=q(x)g(x)+r(x)$ is satisfied in only option (C).

On dividing $f(x)$ by a polynomial $x-1-x^2$, the quotient $q(x)$ and remainder $r(x)$ are $(x-2)$ and $3$ respectively. Then $f(x)$ is

  1. $f(x)=-3x^2-x+7$

  2. $f(x)=-x^3+x^2-x+7$

  3. $f(x)=3x^2-3x+5$

  4. $f(x)=-x^3+3x^2-3x+5$


Correct Option: D
Explanation:

$f(x)=q(x)g(x)+r(x)$

$\therefore f(x)= (x-2)(x-1-x^2)+3$

$\Rightarrow  f(x)= x(x-1-x^2)-2(x-1-x^2)+3$

$=x^2-x-x^3-2x+2+2x^2+3$

$=-x^3+3x^2-3x+5$

On dividing $x^3-3x^2+x+2$ by a polynomial $g(x)$, the quotient and remainder were $(x-2)$ and $(-2x+4)$, respectively. Find $g(x)$.

  1. $2x^2+2x-8$

  2. $x^2+2x-7$

  3. $x^2-x+1$

  4. $2x^2-x+2$


Correct Option: C
Explanation:
By Remainder theorem,
$p(x)=g(x)q(x)+r(x)$

We have, $p(x)=x^3-3x^2+x+2,q(x)=x-2\space and \space r(x)=-2x+4$

$\therefore x^3-3x^2+x+2=g(x)(x-2)+(-2x+4)$

$\Rightarrow x^3-3x^2+x+2+2x-4=g(x)(x-2)$

$\Rightarrow g(x)=\dfrac{x^3-3x^2+3x-2}{(x-2)}=\dfrac{x^3-2x^2-x^2+2x+x-2}{(x-2)}$
                                         
$=\dfrac{[x^2(x-2)-x(x-2)+1(x-2)]}{(x-2)}$

$=\dfrac{(x^2-x+1)(x-2)}{(x-2)}$

$=x^2-x+1$

$\therefore g(x)=x^2-x+1$

Polynomials $p(x), g(x), q(x)$ and $r(x)$, which satisfy the division algorithm and "deg $q(x) = $ deg $ r(x)$", are

  1. $p(x)=2x^2+x; g(x)=2x^2-4$;
    $q(x)=2x-7; r(x)=-x+2$

  2. $p(x)=x^2+x-3; g(x)=x^2+x-1$;
    $q(x)=7; r(x)=-5$

  3. $p(x)=x^2+x; g(x)=x^2-4$;
    $q(x)=2x-1; r(x)=-x-2$

  4. $p(x)=2x^2+2x+8; g(x)=x^2+x+9$;
    $q(x)=2; r(x)=-10$


Correct Option: D
Explanation:

according to division algorithm $p(x)=q(x)g(x)+r(x)$

degree of $q(x)$ is equal to $r(x)$ in all options.
only (D) option satisfies $p(x)=q(x)g(x)+r(x)$
$g(x)q(x)=2(x^2+x+9)=2x^2+2x+18=p(x)+10=p(x)-r(x)$ hence $p(x)=q(x)g(x)+r(x)$ in (D) satisfies division algorithm

On dividing $f(x)=2x^5+3x^4+4x^3+4x^2+3x+2$ by a polynomial $g(x)$, where $g(x)=x^3+x^2+x+1$, the quotient obtained as $2x^2+x+1$. Find the remainder $r(x)$.

  1. $r(x)=7x^3+x^2-1$

  2. $r(x)=3x^2+2x+1$

  3. $r(x)=x-2$

  4. $r(x)=x+1$


Correct Option: D
Explanation:

By remainder theorem,
$f(x)=q(x)g(x)+r(x)$
$\therefore 2x^5+3x^4+4x^3+4x^2+3x+2=(2x^2+x+1)(x^3+x^2+x+1)+r(x)$
$=2x^2(x^3+x^2+x+1)+x(x^3+x^2+x+1)+1(x^3+x^2+x+1)+r(x)$
$=2x^5+2x^4+2x^3+2x^2+x^4+x^3+x^2+x+x^3+x^2+x+1+r(x)$
$=2x^5+3x^4+4x^3+4x^2+2x+1+r(x)$
$r(x)=x+1$

Polynomials $p(x), g(x), q(x)$ and $r(x)$, which satisfy the division algorithm and "deg $p(x) = $ deg $q(x)$" are

  1. $p(x)=2x^2+2x+8, g(x)=4x+1$;
    $q(x)=x^2; r(x)=1$

  2. $p(x)=2x^2+2x+8, g(x)=5$;
    $q(x)=4; r(x)=4x-1$

  3. $p(x)=2x^2+2x+8, g(x)=2$;
    $q(x)=x^2+x+4; r(x)=0$

  4. $p(x)=x^2+x+3, g(x)=2x+3$;
    $q(x)=2x^2+x; r(x)=3x-2$


Correct Option: C
Explanation:

degree of $p(x)$ and $q(x)$ are equal in (A),(C),(D)

according to division algorithm, $p(x)=q(x)g(x)+r(x)$
in option (C), $g(x)q(x)=2(x^2+x+4)=2x^2+2x+8+0=g(x)q(x)+r(x)=p(x)$
hence option (C) is correct answer.

What should be added to $8x^4+14x^3-2x^2+7x-8$ so that the resulting polynomial is exactly divisible by $4x^2+3x-2$?

  1. $10-14x$

  2. $4x-10$

  3. $3x-5$

  4. $5-3x$


Correct Option: A
Explanation:

$4x^2+3x-2)\overline {8x^4+14x^3-2x^2+7x-8}$ ( $2x^2+2x-1$
                          $\underline {\underset {-}{8}x^4\underset {-}{+}6x^3\underset {+}{-}4x^2}$
                                     $8x^3+2x^2+7x-8$
                                     $\underline {\underset {-}{8}x^3\underset {-}{+}6x^2\underset {+}{-}4x}$
                                             $-4x^2+11x-8$
                                             $\underline {\underset {+}{-}4x^2\underset {+}{-}3x\underset {-}{+}2}$
                                                          $14x-10$
We have to add $10-14x$ so that $8x^4+14x^3-2x^2+7x-8$ is completely divisible by $4x^2+3x-2$.

Check whether the first polynomial is a factor of the second polynomial by applying the division algorithm. $x^3-3x+1, x^5-4x^3+x^2+3x+1$

  1. Yes

  2. No

  3. Ambiguous

  4. Data insufficient


Correct Option: B
Explanation:

$x^3-3x+1)\overline {x^5-4x^3+x^2+3x+1}$($x^2-1$
                         $\underset {-}{x^5}\underset {+}{-}3x^3\underset {-}{+}x^2$
                         $\overline {-x^3+3x+1}$
                         $\underline {\underset {+}{-}x^3\underset {-}{+}3x\underset {+}{-}1}$
                                              $2$
Since remainder is non-zero.
Therfore,$x^3-3x+1$ is not a factor of $x^5-4x^3+x^2+3x+1$

If the polynomial $f(x)=x^4-6x^3+16x^2-25x+10$ is divided by another polynomial $x^2-2x+k$, the remainder comes out to be $(x+a)$, then values of $k$ and $a$ are

  1. $k=-2$ & $a=4$

  2. $k=5$ & $a=-5$

  3. $k=-3$ & $a=-7$

  4. None of these


Correct Option: B
Explanation:

$f(x)=$ is divided by another polynomial
$x^2-2x+k)\overline {x^4-6x^3+16x^2-25x+10}(x^2-4x+(8-k)$
                       $\underline {\underset {-}{x^4}\underset {+}{-2x^3}\underset{-}{+}kx^2}$
                       $-4x^3+(16-k)x^2-25x+10$
                       $\underline {\underset {-}{-4x^3}\underset {-}{+8x^2}                      \underset{+}{-}4kx}$
                       $(8-k)x^2+(4k-25)x+10$
                       $\underline {\underset {-}(8-k)x^2+\underset {-}(2k-16)x\underset{-}{+}(8k-k^2)}$
                       $(2k-9)x+(k^2-8k+10)$
But remainder is given $x+a$
$\therefore x+a=(2k-9)x+(k^2-8k+10)$
On equating coefficient, we get
$2k-9=1\Rightarrow k=5$
and $a=k^2-8k+10\Rightarrow a=25-40+10=-5$
Hence, $k=5,a=-5$

Find the value of $b$ for which the polynomial $2x^3+9x^2-x-b$ is exactly divisible by $2x+3$?

  1. $15$

  2. $-15$

  3. $10$

  4. $-10$


Correct Option: A
Explanation:

Since $2x+3$ is a factor of the polynomial $p\left(x\right)=2x^3+9x^2-x-b$
Therefore, by Factor theorem $p\left(-\dfrac32\right)=0$
$\Rightarrow 2\left(-\dfrac32\right)^3+9\left(-\dfrac32\right)^2-\left(-\dfrac32\right)-b=0$

$\Rightarrow -\dfrac{27}4+\dfrac{81}4+\dfrac32-b=0$

$\Rightarrow \dfrac{-27+81+6}4-b=0\Rightarrow b=\dfrac{60}4=15$

$\therefore \space b=15$

What must be subtracted from or added to $8x^4+14x^3-2x^2+8x-12$ so that it may be exactly divisible by $4x^2+3x-2$?

  1. $15x-14$

  2. $3x-14$

  3. $-15x+14$

  4. $-3x+14$


Correct Option: C
Explanation:

$4x^2+3x-2)\overline {8x^4+14x^3-2x^2+8x-12}$ ( $2x^2+2x-1$
                            $\underline {\underset {-}{8}x^4\underset {-}{+}6x^3\underset {+}{-}4x^2}$
                            $8x^3+2x^2+8x-12$
                            $\underline {\underset {-}{8}x^3\underset {-}{+}6x^2\underset {+}{-}4x}$
                            $-4x^2+12x-12$
                            $\underline {\underset {+}{-}4x^2\underset {+}{-}3x\underset {-}{+}2}$
                                        $15x-14$

$\therefore$ The expression that must be subtracted is $15x-14$
and the expression that must be added is $-(15x-14)=-15x+14$

$\displaystyle \left( { 3x }^{ 2 }-x \right) \div \left( -x \right) $ is equal to

  1. $\displaystyle 3x+1$

  2. $\displaystyle -3x-1$

  3. $\displaystyle -3x+1$

  4. $\displaystyle 3x-1$


Correct Option: C
Explanation:

$\displaystyle \left( { 3x }^{ 2 }-x \right) \div \left( -x \right)$

By separating denominators, we get
$  =\dfrac { 3{ x }^{ 2 } }{ -x } +\dfrac { \left( -x \right)  }{ \left( -x \right)  } =-3x+1$
Hence, final result after given operation is $-3x+1$.

A polynomial when divided by $\displaystyle \left ( x-6 \right )$ gives a quotient $\displaystyle x^{2}+2x-13$ and leaves a remainder $-8$. Then polynomial is

  1. $\displaystyle x^{3}+4x^{2}+25x-78$

  2. $\displaystyle x^{3}-4x^{2}-25x+70$

  3. $\displaystyle x^{3}-4x^{2}-25x-70$

  4. $\displaystyle x^{3}+4x^{2}-25x+78$


Correct Option: B
Explanation:
Let $P$ be the polynomial. If $P$ is divided by $(x-6)$ then it leaves a remainder $-8$ and gives a quotient $x^2+2x-13$. Therefore, 

$\cfrac { P }{ x-6 } ={ x }^{ 2 }+2x-13-\cfrac { 8 }{ x-6 } \\ \Rightarrow P=(x-6)({ x }^{ 2 }+2x-13)-\frac { 8(x-6) }{ x-6 } \\ \Rightarrow P={ x }^{ 3 }+2{ x }^{ 2 }-13x-6{ x }^{ 2 }-12x+78-8\\ \Rightarrow P={ x }^{ 3 }-4{ x }^{ 2 }-25x+70$

Hence, the polynomial is $x^3-4x^2-25x+70$.

The expression that should be subtracted from $\displaystyle 4x^{4}-2x^{3}-6x^{2}+x-5$ so that is may be exactly divisible by $\displaystyle 2x^{2}+x-2$ is

  1. $\displaystyle 3x+5$

  2. $\displaystyle -3x-5$

  3. $\displaystyle -3x+5$

  4. $\displaystyle 3x-5$


Correct Option: B
Explanation:
 $2x^2-2x$
$2x^2+x-2$  $4x^4-2x^3-6x^2+x-5$ $4x^4+2x^3-4x^2$
        $-4x^3-2x^2+x$      $-4x^3-2x^2+4x$
                             $-3x-5$

Thus, $-3x-5$ must be subtracted to make it exactly divisble

For a polynomial, dividend is $\displaystyle x^{4}+4x-2x^{2}+x^{3}-10$, quotient is $\displaystyle x^{2}+3x-3x^{2}+4x+12$ and remainder is $14$, then divisor is equal to

  1. $\displaystyle x^{2}+2$

  2. $\displaystyle x^{2}-2$

  3. $\displaystyle x+2$

  4. None of these


Correct Option: D
Explanation:

$p(x)=x^4+4x-2x^2+x^3-10$ and $q(x)=x^2+3x-3x^2+4x+12$ and $r(x)=14$

$p(x)-14$ is not divisible by $(x^2+2),(x^2-2),(x+2)$  
hence correct option is (D)

If $\displaystyle \left ( x^{2}+4x-21 \right )$ is divided by  $x + 7$  then the quotient is

  1. $\displaystyle x+3$

  2. $\displaystyle x-3$

  3. $\displaystyle x^{2}-2$

  4. $\displaystyle x-4$


Correct Option: B
Explanation:

Consider the polynomial $f(x)=x^2+4x-21$ and factorise it as follows:


$f(x)=x^2+4x-21=(x^2+4x+4)-21-4=(x+2)^2-25=(x+2)^2-(5)^2=(x+2+5)(x+2-5)$
$=(x+7)(x-3)$

Therefore, $f(x)=(x+1)(x-1)(x-3)$

Let $g(x)=x+7$

Now divide $f(x)$ by $g(x)$ to get $q(x)$:

$q(x)=\frac { (x+7)(x-3) }{ (x+7) } =x-3$

Hence, the quotient is $x-3$.

If $\displaystyle f(x)=x^{4}-2x^{3}+3x^{2}-ax+b$ is a polynomial such that when it is divided by $( x - 1 )$ and $( x +1)$, the remainders are $5$ and $19 $ respectively, the remainder when $f(x)$ is divisible by $(x -2 ) $ is 

  1. $7$

  2. $8$

  3. $9$

  4. $10$


Correct Option: D
Explanation:

When ${ x }^{ 4 }-{ 2x }^{ 3 }+3{ x }^{ 2 }-ax+b$ is divide by $ x-1,$ remainder is $5.$
So, substituting for $x$ is $1,$ in the above, we get

$5=1-2+3-a+b$ 
$\therefore-a+b=3 $----(1)

When ${ x }^{ 4 }-{ 2x }^{ 3 }+3{ x }^{ 2 }-ax+b$ is divide by $x+1$, remainder is $19.$
So, substituting for $x$ is $-1,$ in the above, we get 
$19=1+2+3+a+b$
$a+b=13 $----(2)
Solving (1) and (2), we get $a=5,b=8$
So polynomial becomes  ${ x }^{ 4 }-{ 2x }^{ 3 }+3{ x }^{ 2 }-5x+8$
The remainder when ${ x }^{ 4 }-{ 2x }^{ 3 }+3{ x }^{ 2 }-5x+8$ is divided by $x-2$ is by plugging in $x$ as $2$ in the given polynomial, we get 

$16-16+12-10+8=10$
so remainder is $10$
So, option D.

When a number is divided by $13$, the remainder is $11$. When the same number is divided by $17$, the remainder is $9$. What is the number ?

  1. $853$

  2. $278$

  3. $349$

  4. $670$


Correct Option: C
Explanation:

$x = 13p + 11$ and $x = 17q + 9$
$\therefore$ $13p + 11 = 17q + 9$
$\therefore$ $17q - 13p = 2$
$\therefore$ q $=\dfrac{2 + 13p}{17}$
The least value of p for which q $=\dfrac{2 + 13p}{17}$ is a whole number is $p = 26$
x $= (13 \times 26 + 11)$
$= (338 + 11)$
$= 349$

On dividing a number by $56$, we get $29$ as remainder. On dividing the same number by $8$, what will be the remainder ?

  1. $4$

  2. $5$

  3. $6$

  4. $7$


Correct Option: B
Explanation:

Applying remainder theorem i.e. $A=bq+r $
where, $b$ = divisior 
$r$ = remainder 
$\therefore A = 56q + 29$
if q = 1 
The no. is $A = 85$
On dividing by $8$, Remainder $(r) =5$

The remainders of polynomial f(x) when divided by x-1, x-2 are 2,3 then the remainder of f(x) when divided by (x-1) (x-2) is

  1. 2x-1

  2. x-1

  3. 2x+1

  4. x+1


Correct Option: D
Explanation:
According to Remainder theorem

$f(x)=(x-1)(x-2) \theta (x)+\gamma (x)$

$\gamma (x)=ax+b$

$f(1)=a+b=2$     $\dots(1)$

$f(2)=2a+b=3$     $\dots(2)$

Subtract $(1)$ from $(2)$

$2a+b-a-b=3-2$

$\Rightarrow a=1$ substitute in $(1)$

$b=2-1=1$

$\therefore\ a=b=1$

So, $\gamma (x)=x+1$

If the remainders of the polynomial f(x) when divided by x+1 and x-1 are 3, 7 then the remainder of f(x) when divided by $(x^{2} -1 )$ is

  1. x + 4

  2. 2x + 3

  3. 2x + 4

  4. 2x + 5


Correct Option: D
Explanation:
According to remainder theorem

$f(x)=\theta (x)(x^{2}-1)+\gamma (x)$

$\gamma (x)=ax+b$

So, $f(x)=\theta (x)(x^{2}-1)+(ax+b)$

$f(-1)=-a+b=3$      $\dots(1)$

$f(1)=a+b=7$         $\dots(2)$

Add $(1)$ and $(2)$

$-a+b+a+b=10\Rightarrow 2b=10\Rightarrow b=5$

substitute it in $(1)$ then

$a=5-b=5-3=2$ 

$\Rightarrow b=5; a=2$

So $\gamma (x)=2x+5$

Given $f(x)$ is a cubic polynomial in $x$. If $f(x)$ is divided by $(x + 3), (x + 4), (x + 5)$ and $(x + 6)$ then it leaves the remainders $0, 0, 4$ and $6$ respectively. Find the remainder when $f(x)$ is divided by $x + 7$.

  1. $0$

  2. $1$

  3. $2$

  4. $3$


Correct Option: A
Explanation:

It is given that $f(x)$ leaves the remainder $0$ if divided by $(x+3)$ and $(x+4)$, which implies that $(x+3)$ and $(x+4)$ are factors of $f(x)$.


Let the other factor be $ax+p$, then $f(x)$ is given by:

$f(x)=(x+3)(x+4)(ax+p)$

Now, it is also given that $f(x)$ leaves the remainders $4$ and $6$ if divided by $(x+5)$ and $(x+6)$, which means that $f(-5)=4$ and $f(-6)=6$.

If $f(-5)=4$, then we have:

$f(x)=(x+3)(x+4)(ax+p)\ \Rightarrow f(-5)=(-5+3)(-5+4)(a(-5)+p)\ \Rightarrow 4=(-2)(-1)(-5a+p)\ \Rightarrow 4=2(-5a+p)\ \Rightarrow -5a+p=2\quad ........(1)$

And if $f(-6)=6$, then we have:

$f(x)=(x+3)(x+4)(ax+p)\ \Rightarrow f(-6)=(-6+3)(-6+4)(a(-6)+p)\ \Rightarrow 6=(-3)(-2)(-6a+p)\ \Rightarrow 6=6(-6a+p)\ \Rightarrow -6a+p=1\quad ........(2)$

Subtract eqn 2 from eqn 1 as follows:

$[-5a-(-6a)]+(p-p)=2-1\ \Rightarrow (-5a+6a)+0=1\ \Rightarrow a=1$

Substitute the value of $a$ in eqn 1:

$(-5\times 1)+p=2\ \Rightarrow -5+p=2\ \Rightarrow p=2+5=7$

Therefore, 

$f(x)=(x+3)(x+4)[(1\times x+7)]\ \Rightarrow f(x)=(x+3)(x+4)(x+7)$

Thus, $f(-7)=0$

Hence, $f(x)$ leaves the remainder $0$ when divided by $x+7$.

Find the remainder when  $-2x^3-2x^2+27x-30$ is divided by $2-x$.

  1. $0$

  2. $1$

  3. $2$

  4. $3$


Correct Option: A
Explanation:
Quotient----->  $2x^2+6x-15$
 -x + 2   $-2x^2-2x^2+27x-30$  $-2x^3+4x^2$
    $-6x^2+27x-30$ $-6x^2+12x$-----------------------------------$15x-30$$15x-30$-------------------0
 
 

$p(x)=(x^2-10x-24)$ , when divided by $x+2$ and $x\neq -2$ gives the quotient $Q$. Find $Q$.

  1. $x -22$

  2. $x-12$

  3. $x+12$

  4. $x+22$


Correct Option: B
Explanation:
 Quotient  $x - 12$
 $x+2$   $x^2-10x-24$  $x^2+2x$$- $   $-$
         $ -12x-24$        $ -12x-24$       $+$        $+$------------------------------                 $ 0$

Using division algorithm method we get the value of $Q = x - 12$.

When $(x^3-2x^2+px-q)$ is divided by $(x^2-2x-3)$, the remainder is $(x-6)$. The values of $p$ and $q$ respectively are ____. 

  1. $-2, 7$

  2. $2, -6$

  3. $-2, 6$

  4. $2, 6$


Correct Option: C
Explanation:

$x^2-2x-3=(x+1)(x-3)$

$x^3-2x^2+px-q-(x-6)=0$ at $x=-1$ and $x=3$.         ($\because (x-6)$ is the remainder)
put $x=-1$,
$-1-2-p-q+1+6=0\ \Rightarrow p+q=4\dots eqn (1)$
Now, put $x=3$
$27-18+3p-q-3+6=0\ \Rightarrow 3p-q=-12\dots eqn (2)$
Add equation 1 and 2, we get
$4p=-8\Rightarrow p=-2$
 and $q=4-p=6$

When a polynomial $P(x)$ is divided by $x, (x - 2)$ and $(x - 3)$, remainders are $1$, $3$ and $2$ respectively. the same polynomial is divided by $x(x - 2)(x - 3)$, the remainder is $ax^2 + bx + c$, then the value of $c$ is

  1. $-3$

  2. $-2$

  3. $6$

  4. $1$


Correct Option: D
Explanation:

According to factor theorem

$p(x)=xq(x)+1$
$P(x)=(x-2)q'(x)+3$
$P(x)=(x-3)q''(x)+2$
So, At $x=0 , P(0)=1$
At $x=2 , P(2)=3$
At $x=3 , P(3)=2$
Now when the polynomial $P(x)$ is divided by $(x-2)(x-3)x$ the remainder must have the degree less than $3$ . that is the remainder will be of the form $ax^2+bx+c$
$\implies P(x)=x(x-2)(x-3)q''''(x)+ax^2+bx+c$
So, $P(0) =1=a(0)^2+b(0)+c\implies c=1$

The quotient and remainder when $x^{2002}$ $- 2001$ is divided by $x^{91}$ are 

  1. $x^{91 \times 22}, 2001$

  2. $x^{91}, 2001$

  3. $x^{91\times 21}, -2001$

  4. $x^9, -2001$


Correct Option: C
Explanation:

$2002 = 91 \times 22$

$\therefore x^{2002} = x^{91 \times 22}$
$\therefore$ $x^{2002} - 2001$ $=$ $x^{91} \times (x^{91 \times 21}) - 2001$

When $x^{91} \times (x^{91 \times 21}) - 2001$ is divided by $x^{91}$, then
Quotient $= x^{91 \times 21}$ 
And 
Remainder $= -2001$

Which of the following given options is/ are correct?
If $p(x)=q(x)g(x)+r(x)$ (By Division Algorithm) where p(x), g(x) are any two polynomials with $g(x)\neq 0$, then

  1. $r(x)=0$ always

  2. degree of r(x)< degree of g(x) always

  3. either $r(x)=0$ or degree of r(x)< degree of g(x)

  4. $r(x)=g(x)$


Correct Option: C
Explanation:

(a) If p(x) is not divisible by g(x), then $r(x)\neq 0 \therefore$ (a) is not true
(b) If p(x) is divisible by g(x), the $r(x)=0$ for all x i.e., r(x) is zero polynomial whose degree is not defined.
$\therefore $(b) is not true
(c) is clearly true [$\because$ division algorithm rule]
Since degree of $r(x)<$ degree of g(x) or $r(x)=0$, but $g(x)\neq 0$.
(d) $\therefore r(x)=g(x)$ is not true.

A polynomial $f(x)$ with rational coefficient leaves reminder $15$, when divided by $(x-3)$ and remainder $2x+1$, when divided by $(x-1)^{2}$. If $p$ is coefficient of $x$ of its remainder which will come out if $f(x)$ is divided by $(x-3)(x-1)^{2}$ then find $p$.

  1. $-2$

  2. $-1$

  3. $1$

  4. $6$


Correct Option: A

The remainder obtained when the polynomial $1+x+x^ {3}+x^ {9}+x^ {27}+x^ {81}+x^ {243}$ is divisible by $x-1$ is

  1. $3$

  2. $5$

  3. $7$

  4. $11$


Correct Option: C
Explanation:

$p{\left( x \right)} = 1 + x + {x}^{3} + {x}^{9} + {x}^{27} + {x}^{81} + {x}^{243}$

Let $q{\left( x \right)}$ be the quotient when $P{\left( x \right)}$ divided by $\left( x - 1 \right)$.
Therefore,
$P{\left( x \right)} = \left( x - 1 \right) \cdot q{\left( x \right)} + A$
$P{\left( 1 \right)} = \left( 1 - 1 \right) \cdot q {\left( x \right)} + A$
$7 = 0 + A$
$A = 7$
Hence the remainder is $7$.

If $A=2x^{3}+5x^{2}+4x+1$ and $B=2x^{2}+3x+1$, then find the quotient from the following four option, when A is divided by B.

  1. $x-1$

  2. $x+1$

  3. $2x+1$

  4. $2x-1$


Correct Option: B
Explanation:


$\dfrac{A}{B} = \dfrac{2x^{3}+5x^{2}+4x+1}{2x^{2}+3x+1}$

$=\dfrac{2x^{3}+(3x^{2}+2x^{2})+(x+3x)+1}{2x^{2}+2x+1}$

$=\dfrac{(2x^{3}+3x^{2}+x)+(2x^{2}+3x+1)}{2x^{2}+3x+1}$

$=\dfrac{(2x^{2}+3x+1)(x+1)}{2x^{2}+3x+1}=x+1$

Evaluate: $\displaystyle \frac{a^3\, +\, b^3\, +\, c^3\, -\, 3abc}{a^2\, +\, b^2\, +\, c^2\, -\, ab\, -\, bc\, -\, ca}$

  1. $0$

  2. $a + b + c$

  3. $1$

  4. None of these


Correct Option: B
Explanation:

$\cfrac { { a }^{ 3 }+{ b }^{ 3 }+{ c }^{ 3 }-3abc }{ { a }^{ 2 }+{ b }^{ 2 }+{ c }^{ 2 }-ab-bc-ca } \ =\cfrac { \left( a+b+c \right) \left( { a }^{ 2 }+{ b }^{ 2 }+{ c }^{ 2 }-ab-bc-ca \right)  }{ { (a }^{ 2 }+{ b }^{ 2 }+{ c }^{ 2 }-ab-bc-ca) } \ =a+b+c$.

If x + 2 and x-1 are the factors of $x^3 + 10x^2+mx + n$, then the values of m and n are respectively

  1. 5 and -3

  2. 17 and -8

  3. 7 and-18

  4. 23 and -19


Correct Option: C
Explanation:

Here, $x + 2$ is a factor of $x^3 + 10x^2 + mx + n$
$x =-2$
$(-2)^3 + 10(-2)^2 + m(-2) + n =0$
$ -8 + 40 = 2m - n $
$2m -n = 32$                     .....(i)
Again, $x-1 $ is a factor of $x^3 + 10x^2 + mx + n$
$x =1$
$1+10+m+n=0$
$m + n =-11$                    .....(ii)
Adding (i) and (ii). we get,
$3m = 21$
$m=7$
By putting m in (i). we get,
$2(7) - n = 32 $
$ - n = 18 $
   $n = -18$

Option C is correct.

The expression $2x^3 + ax^2 + bx +3$, where a and b are constants, has a factor of x-1 and leaves a remainder of 15 when divided by x+2. Find the value of a and b respectively.

  1. $-3, 8$

  2. $3,-8$

  3. $-3,-8$

  4. $3, 8$


Correct Option: B
Explanation:

f(x)=$ 2x^3+ax^2-bx+3$ 
At x=2 
f(2)=15 
f(1)=0 
f(x)=$ 2x^3+ax^2-bx+3$ 
f(1)= 2+a-b+3=0 
a-b+5=0......A 
f(x)= $2x^3+ax^2-bx+3$ 
f(2)=$ 2(2^3)+a(2^2)-2b+3=15 $
4a-2b=-4 
Multiply A by 2 and subtract from above equation 
4a-2b=-4 
2a-2b+10=0 
2a-10=-4 
2a= 6 
a=3 
From A 
3-b+5=0 
8-b=0 
b=8 
So a=3 and b=8

If $(x^{3} + 5x^{2} + 10k)$ leaves remainder $-2x$ when divided by $(x^{2} + 2)$, then what is the value of k?

  1. $-2$

  2. $-1$

  3. $1$

  4. $2$


Correct Option: C
Explanation:

$x^{3} + 5x^{2} + 10k$
$= (x^{2} + 2)(x + 5) + 10k - 2x - 10$
$\Rightarrow 10k - 2x - 10 = -2x$
$\Rightarrow 10k - 10 = 0$ or $k = 1$.

The polynomial $f(x)={ x }^{ 4 }-2{ x }^{ 3 }+3{ x }^{ 2 }-ax+b$ when divided by $(x-1)$ and $(x+1)$ leaves the remainders $5$ and $19$ respectively. Find the values of $a$ and $b$. Hence, find the remainder when $f(x)$ is divided by $(x-2)$

  1. $a=6,b=8$ and $remainder=10$

  2. $a=5,b=8$ and $remainder=10$

  3. $a=5,b=7$ and $remainder=10$

  4. None of these


Correct Option: B
Explanation:

    by remainder theorem,

      $f(1)=5$ $and$ $f(-1)=14$

     $\therefore 1-2+3-a+b=5\Rightarrow b-a=3$  and
     $1+2+3+a+b=19\Rightarrow a+b=13$ 
      $b=8$ $a=5$ 
       and remainder when $f(x)$ is divided by $(x-2)$ is
         $f(2)=16-16+12-10+8=10$

$32x^{10}-33x^{5}+1$ is divisible by 

  1. $x-1$

  2. $x-2$

  3. $x-3$

  4. $x-4$


Correct Option: A
Explanation:
$32x^{10}- 33 x^5 +1=0$
Let $m = x^5$
$\Rightarrow 32 m^2 - 33 m + 1 =0$
$\Rightarrow (32 m -1)(m-1)=0$
$\Rightarrow (32x^5-1)(x^5-1)=0$
$\therefore 32x^{10}- 33 x^5 +1=(32x^5-1)(x^5-1)$

$x^n-y^n$ is always divisible by $(x-y)$

$\therefore \,32x^{10}- 33 x^5 +1$ is divisible by $(x-1)$

The remainder, when $({ 15 }^{ 23 }+{ 23 }^{ 23 })$ is divided by $19$, is 

  1. $4$

  2. $17$

  3. $23$

  4. $0$


Correct Option: D
Explanation:
Given, $(15)^{23}+(23)^{23}$

$=(19-4)^{23}+(19+4)^{23}$

$\Rightarrow $ In bino  expansion  of above expression the term containing  19 well be cancelled 
as they disable by 19 then remaining term are 

$\Rightarrow  (-4)^{23}+(4)^{23}=0$

Therefore the remainder is exactly  zero .

If $(x^{100} + 2x^{99} + K)$ is exactly divisible by $(x + 1)$, find the value of 'K'

  1. $1$

  2. $2$

  3. $-2$

  4. $-3$


Correct Option: A
Explanation:

$x^{100}+2x^{99}+k$ is exactly divisible by $(x+1)$

$\therefore x=-1$ is the root of $x^{100}+2x^{99}+k$
$\Rightarrow (-1)^{100}+2(-1)^{99}+k=0$ 
$\Rightarrow 1-2+k=0$ 
$\Rightarrow \boxed{k=1}$

The sum of the digits of a 3 digit number is subtracted from the number. The resulting number is always.

  1. Divisible by 6

  2. Not divisible by 6

  3. Divisible by 9

  4. Not divisible by 9


Correct Option: C
Explanation:

Let the no. be $xyz$ sum of digit is $(x+y+z)$ 

as $xyz=100x+10y+z$
then $xyz-(x+y+z)=99x+9y$  
$\therefore $ $\boxed{Always\, divisible\, by\, 9}$

The remainder when the polynomial $1+x^2+x^4+x^6+....+x^{22}$ is divided by $1+x+x^2+x^3+....+x^{11}$ is?

  1. $0$

  2. $2$

  3. $1+x^2+x^4+...+x^{10}$

  4. $2(1+x^2+x^4+....+x^{10})$


Correct Option: D
Explanation:
$ \left( \sum _{n=0}^N x^{n} \right) = \left ( \dfrac{x^{N+1}-1}{x-1} \right ) $

$ \Rightarrow Dividend = \left ( \dfrac{x^{24}-1}{x^{2}-1} \right ) $
$ Divisor = \left ( \dfrac{x^{12}-1}{x-1} \right ) $

Now,
$ \left ( \dfrac{x^{24}-1}{x^{2}-1} \right )  = \left ( \dfrac{x^{12}-1}{x-1} \right ) \left ( \dfrac{x^{12}-1+2}{x+1} \right ) = \left ( \dfrac{\left ( x^{12}-1 \right )^{2}}{x^{2}-1} \right ) + 2\left ( \dfrac{x^{12}-1}{x^{2}-1} \right ) $

$ \Rightarrow Remainder = 2\left ( 1+x^{2}+x^{4}...+x^{10} \right ) $

If the polynomial $x^{19}+x^{17}+x^{13}+x^{11}+x^7+x^5+x^3$ is divided by $(x^2+1)$, then the remainder is:

  1. $1$

  2. $x^2+4$

  3. $-x$

  4. $x$


Correct Option: C

The decimal representation of $2005!$ ends with $m$ zeroes then $m=$

  1. $500$

  2. $501$

  3. $502$

  4. $499$


Correct Option: A
Explanation:
We know that a number gets a zero at the end of it if the number has 10 as a factor.
  
So I need to find out how many times 10 is a factor in the expansion of 23!.

But since 5×2 = 10, I need to account for all the products of 5 and 2.
 Looking at the factors in the above expansion, there are many more numbers that are multiples of 2 (2, 4, 6, 8, 10, 12, 14,...) than are multiples of 5 (5, 10, 15,...). 
That is, if I take all the numbers with 5 as a factor, I'll have way more than enough even numbers to pair with them to get factors of 10 (and another trailing zero on my factorial).
No. of multiples of 5 between 1 and 2005!= 401
No. of multiples of 25= 80
no. of multiples of 125=16
no. of multiples of 625=3
Hence total multiples of 5 and hence 10 are 500.
There are 500 zeroes in the end of 2005!.

What will be the Remainder when $3x^{3} - 2x^{2} - 7x + 6$ is divided by $x + 1$?

  1. $4$

  2. $5$

  3. $8$

  4. $0$


Correct Option: C
Explanation:

Let $f(x)=3x^3-2x^2-7x+6$

To find out reminder when equation is divided by $x+1$ we should calculate $f(-1)$
$f(-1)=3(-1)^3-2(-1)^3-7(-1)+6$
            $=13-5=8$

A body falling from rest under gravity passes a certain point $P$.It was a distance of $400m$ from P and $4$ sec prior to passage through $P$ If $g=10m/sec^2$,then the height above the point $"P"$ from where the body began to fall is ?

  1. $900m$

  2. $320m$

  3. $680m$

  4. $720m$


Correct Option: B
Explanation:
Distance travelled $=400\ m$.
Time$=4\ sec$
$B=10m/s^{2}$
$s=ut+1/2 at^{2}$
$400=4u-1/2\times 10\times 16\times 5$
$400=4u-80$
$4u=480$
$u=120$
At highest point
$V=0$
${u}^{2}=2\times g\times h$
$120\times 120=2\times 10\times h$
$h=720$
This height is from $400\ mtr$ below $P$ 
So height above $P$ is $720-400=320\ mtrs$

The remainder when $x^3 + 4x^2 - 7x + 6$ is divided by $(x - 1)$ is

  1. $4$

  2. $0$

  3. $-4$

  4. $3$


Correct Option: A
Explanation:

Let $f\left( x \right) =x^{ 3 }+4x^{ 2 }-7x+6$
As $f\left( x \right) $ is divided by $x-1$, substituting $x=1$ in $f\left( x \right) $ we get
$f\left( 1 \right) =1^{ 3 }+4\cdot1^{ 2 }-7\cdot1+6=4$
Hence, $4$ is the remainder.

What will be the Quotient when $4x^{3} - 8x^{2} - x + 5$ is divided by $2x - 1$?

  1. $2x^{2} - 3x - 2$

  2. $3x^{2} - 6x - 2$

  3. $4x^{2} - 6x +4$

  4. $2x^{2} - 6x - 2$


Correct Option: A
Explanation:

Given: equation $4x^3-8x^2-x+5$

To find the quotient when divided by $2x-1$
Sol: $2x-1)\overline{4x^3-8x^2-x+5}(2x^2-3x-2)\\quad\quad \quad 4x^3-2x^2\\quad\quad\quad \overline{\quad \quad -6x^2-x}\\quad\quad\quad\quad\quad- 6x^2+3x\\quad\quad\quad\overline{\quad\quad\quad\quad \quad -4x+5}\\quad\quad\quad\quad\quad\quad \quad \quad- 4x+2\\quad\quad\quad\quad\overline {\quad\quad\quad\quad\quad\quad\quad 3} $

- Hide questions